Howdy, Stranger!

It looks like you're new here. If you want to get involved, click one of these buttons!

RC Annotation Method and BR (Most Authoritarian Rulers Passage pt.2)

bbutlerbbutler Inactive ⭐
edited February 2016 in Reading Comprehension 401 karma
Hey guys after watching Nicole Hopkins' webinar on RC Methods and Jimmy Quicksilver's webinar on RC Question Types and Tips I thought it would be helpful to start sharing our notes/annotations for each passage because each of us reads a different way and we all see different things. I want to be clear that I’m no expert by any means and this is just the way I personally annotate that is a combination of Nicole Hopkins’s “Toolbox” method and JY’s Memory Method. Also note that I’m doing RC a slightly similar way to Pacifico’s Fool Proof method in that I’m doing the passage 2x one after the other and then once again the following day. While time consuming this definitely is allowing me to read more efficiently for structure and see the similarities in each passage which I know will help me in the long run. This is part two which includes my breakdown of all of the questions in a similar manner to which I did when I was tutored by Nicole Hopkins and how we broke everything down.

I hope it helps and I look forward to seeing what you guys come up with.

Questions:
1) What is the Main Point of the passage

MP Question: “Main Point of the passage”

What we’re looking for:
• We’re looking for something to basically re-phrase the entire first paragraph, something that says “authoritarian rulers are forced to undertake democratic reform, if they want to stay in power, due to various changes and mobilizations in society.

Answer Choices:

A) Wrong: Being this far in the curriculum when you see the word “only” you should remember from your logical indicators that it immediately makes whatever follows the necessary condition. So we can rephrase this answer choice to be read as “If authoritarian rulers undertake democratic reform then the national’s economic and social power bases will slow economic growth and disrupt social order until such reforms are instituted.” For this answer choice to be right there would have to be evidence in the passage of undertaking democratic reform to be a sufficient condition and the only thing that we could infer if authoritarian rulers undertook democratic reform would be that they would be able to hold onto some of their power. This answer choice plays the trap of reversing the sufficient and necessary conditions, if it were reversed then one could argue that if economic and social based slow economic growth and disrupt social order then we will undertake democratic reform.

Why You would accidentally choose this:
• This answer choice holds a lot of the same words that are used in the passage and if you didn’t see the logical indicator or interpreted it the wrong way then this answer choice could seem attractive. However don’t fall for this, know your logical indicators and read the answer choice carefully and you won’t fall into trouble.

B) Wrong: When reading this answer choice it sounds really good to start, because the author does state that these Authoritarian regimes do ensure their own destruction. However, it isn’t for the reason that is listed here, they don’t talk about opposition groups to build support among the wealthy to lead the support away. The reason is in lines (51-57), the more success and stability the authoritarian regime has, the more time it gives for citizens to reflect on the circumstances in which they live which brings about these changes.

Why You would accidentally choose this:
• You could fall for the trap if you assumed that the opposition group referred back to the “privileged people” talked about in the 3rd paragraph. From there you could make a connection that this was talking about the second change that contributed to making it impossible for authoritarian rulers to hold onto their power. However, that still doesn’t actually answer the question that we’re after and even though this seems logical we have to actually answer the question that is asked

C) Wrong: This is similar to “B” in that it sounds really good to start off however it misses the mark when it says “success at generating economic growth and stability will be short lived”. This completely factually inaccurate the only thing that is short lived is the authoritarian regime but none of the reasons given were because the economic growth and stability were going to be short lived. In lines (51-57) you can see that actually the more success the regime has the short lived it will be because it gives citizens more time to reflect on their current situation

Why you would accidentally choose this:
• You would think this is right if you didn’t pick up the distinction that when the answer choice refers to “short lived” it isn’t talking about the regime it’s talking about the economic growth and stability. If you didn’t pick up on that then you could piece together that economic polices did alienate the economic power base, based off of the information in paragraphs 3 and 4. However, you have to read carefully so you don’t fall for the traps that the test makers set for you

D) Wrong: This answer choice is wrong because it says untenable (otherwise known as unattainable) and that is factually inaccurate. The point of the passage is that authoritarian regimes are not SUSTAINABLE however they are attainable. There are lots of traps here in this answer choice. First you need to see that the answer choice talks about authoritarian principles whereas the passage talks about the regimes specifically so that should immediately raise your suspicion. Then we already pointed out that authoritarian regimes are actually attainable and the main point is that they are not sustainable. Then finally if you somehow made it that far there is nothing in the passage that says that the reason why they’re not sustainable is because they require a degree of social and economic stability that only a democratic institution can create. As we’ve seen in lines (51-57) an authoritarian regime can have success and that ultimately leads to its demise.

Why would accidentally choose this:
• If you immediately read sustainable instead of untenable and then circled and moved on. Time is definitely not on your side but make sure you read all of the answer choices and read the rest of the information because many answer choices will have pieces that are true but only one will be true in all aspects.

E) Correct: This is exactly what we’re looking for. Let’s break down this answer choice. “Authoritarian rulers who instituted democratic reforms” lets us know that we’re talking about the same subject matter. Then “are compelled to do so because authoritarian rule tends to bring about various changes in society” refers to the 3 changes that were talked about throughout the passage. Finally, “that eventually necessitate corresponding political changes” refers to undergoing democratic reform.

Why you would accidentally NOT choose this:
• This answer choice isn’t a “sexy” answer choice. It leaves something to be desired because you have to connect the dots on the “various changes in society” referring to the changes talked about in the passage, along with “necessitating corresponding political changes” referring to undergoing democratic reform. This is how a lot of the correct answer choices are going to be written in RC, so don’t not choose an answer choice just because it doesn’t immediately come out and say exactly what you want, you’re going to have to work for it on some of the questions.

2) The author’s attitude toward authoritarian regimes is most accurately described as which of the following:

“Author attitude question”

What we’re looking for:
• The answer choices could go a multitude of ways; we could either say something along the lines as sympathetic because their success brings their own demise (51-57) or something along the lines of confident that authoritarian regimes will undergo democratic reform (56-60).

Answer Choices:
A) Wrong: This is really never talked about in the passage but it doesn’t seem correct when compared to our anticipated answer choices. The author seems to have 2 polar opposite attitudes regarding authoritarian regimes and “uncertain” doesn’t qualify as one of them.

Why would you accidentally choose this:
• If you weren’t looking for the author’s attitude then you could justify this answer choice by bringing in outside information because the author does seem to be neutral towards whether this is progress or not but you have to actually answer the question that is given, and this answer choice doesn’t do that.

B) Wrong: This hints at what we where going after because it does have the elements that authoritarian rulers do bring about their own demise. However, the author doesn’t address the motives of the rulers, he doesn’t explicitly say something like “Based on the past authoritarian regimes success, rulers should reconsider their views”

Why you would accidentally choose this:
• If you just saw “tendency to bring about their own demise” then this answer choice would be extremely attractive. You have to read all of the answer choice and not just parts because parts could be right but if the entirety of the answer choice isn’t right then the answer choice isn’t right.

C) Correct: This is exactly what we’re looking for and it matches out anticipated answer choice. We know that the author is confident that democratic forms of government will replace authoritarian regimes because he says so in the last sentence of the passage. “The more astute authoritarian rulers recognize that their only hope of maintaining some power in the future is with democratic political changes”

Why you would accidentally NOT choose this answer choice:
• If you fell for the trap of answer choice “B” then you wouldn’t have read this answer choice and you could get the question wrong. Also if you didn’t pick up what the last sentence of the passage was saying then you also would miss this answer choice. Read all of the answer choices and read carefully when reading the passage.

D) Wrong: Nowhere in the passage does the author say that authoritarian rule constitutes an “unjust form of government”, and if it’s not in the passage then it can’t be a correct answer choice.

Why you would accidentally choose this answer choice:
• This is one of the oldest tricks in the book for test makers, they want to play with your emotions and make put you in the position to draw off of your outside knowledge and insert your opinion on this information. Based off of the passage it very well could be that you believe that authoritarian rules are an unjust form of government but nothing in the passage states that THE AUTHOR believes that.

E) Wrong: This is extremely factually inaccurate, the author seems to believe that there is no way that authoritarian rulers can retain power without instituting democratic reform. That is the basis of his argument and the main point of the passage.

Why you would accidentally choose this answer choice:
• If you didn’t read this answer choice carefully and read that authoritarian rulers WONT discover ways to retain their power without instituting democratic reforms then you could logically work your way to choose this answer choice. The problem is even if the answer choice were written in that manner I still don’t believe that the author has any “concern” towards the authoritarian rulers I believe he is simply stating an argument but leaves out his personal feelings towards the predicament the authoritarian rulers are in

3) Which of the following titles most completely summarizes the content of the passage?

Title Question: Very similar to MP or a structure question
• Look at how the passage is written and choose the best title that matches the flow of the passage

A) Wrong: The first part of this answer choice is right however the author doesn’t make a dissent against authoritarian regimes. He never uses a paragraph to say that their principles are bad or that they’re crazy because the more success they have the more they dig their own grave, nothing.

Why you would accidentally choose this:
• This would combine a couple traps that you would have when taking the test. Either you didn’t read all of the answer choice because you were trying to save time so you didn’t read, “dissent against” or you did read, “dissent against” and brought in personal information and chose the answer choice. Don’t do either, and don’t fall for the trap.

B) Wrong: Nothing in the passage talks about human rights being abused or anything like that, this one should be an immediate deletion. Also when it says case study I interpret that as an example that is fleshed out throughout multiple paragraphs, the author used multiple different examples but he didn’t have one single overarching example that everything fit under.

Why you would accidentally choose this:
• Again if you brought in outside information that you personally felt towards the authoritarian regimes then you could realistically get to this answer choice but it still doesn’t make it right. Also if you saw authoritarian regimes and just circled it and moved on then you also could choose this, don’t fall for the traps!

C) Wrong: This is a TRAP ANSWER CHOICE because it has all of the right elements. However you have to think about what this is actually saying, this title would be right if the passage was referring to exactly how the democratic reforms were going to be laid out. There would be specific examples of what reforms would be done, maybe examples of democratic reforms in the past. Our passage however is simply saying that democratic reforms are going to happen and why, not exactly what the reforms are going to be.

Why you would accidentally choose this:
• If you fell for the trap of thinking that this was exactly what we were looking for because it had all of the right elements then you’d choose this answer choice. Read all of the answer choice and then ask yourself were their specific strategies/solutions that the passage laid out regarding democratic reform?

D) Correct: This is exactly what we’re looking for. “Why authoritarian regimes compromise” refers to the 3 changes/causes that makes it occur. While “examination of social forces” refers to each paragraph going into detail about each of the changes that are actually causing the reform to occur.

Why you would accidentally NOT choose this:
• If you fell for the trap answer choice “C” then you wouldn’t have gotten this, also if you didn’t feel like this was “strong enough” so you were looking for another answer that was “better” then you might have overlooked this. The answer choices you choose don’t have to jump out right away they just have to be the right answer choice.

E) Wrong: This isn’t the main part of the paragraph because it doesn’t talk about economic instability as the main reason why, as we’ve established throughout the prior questions the more stable the economy and the better the regime is doing, the more likely the regime will fall.

Why you would accidentally choose this:
• If you were running out of time and said that I know the author believes that most of these countries are going to undergo democratic reform, so that makes this subject matter correct. And there were some talks of economics in the passage so this is right. Just because the elements are there doesn’t mean that they’re talked about in the right way. Don’t fall for traps like this because it’s just a mashed potato answer choice.

4) Which of the following most accurately describes the organization of the passage?

Structure Question: “Describes the organization”

What we’re looking for:
• We want an answer choice that states the author states an issue (authoritarian rulers can’t maintain their power unless democratic reform occurs), and then lists the causes of that issue while providing examples, (values and norms shift, economic interests shift, expanding resources, autonomy and self confidence), and then the author reaffirms his position at the end.

Answer Choices:
A) Wrong: Everything in the beginning of this answer choice is right but when it gets to “relative importance” that’s when it’s done. Also afterwards the answer states, “possibility of alternate causes is considered and rejected” which also doesn’t occur.

Why you would accidentally choose this:
• If you didn’t read carefully and just breezed through this. These types of questions, the answer choices are usually extremely similar to one another and so one minor detail is how they differentiate from each other. Read carefully and read the entire answer choice.

B) Correct: This is exactly what we’re looking for. A political phenomenon (authoritarian rulers can’t maintain their power unless democratic reform occurs) is linked to a general set of causes (values and norms shift, economic interests shift, expanding resources, autonomy and self confidence), an explanation of each is given, then the causal relationship is elaborated and confirmed (author reaffirms his position at the end).

Why you would accidentally NOT choose this:
• If you didn’t label the passage for structure then when you get to the answer choices they all will sound similar. So you could miss this because you marked it as a maybe and then fell for another answer choice when you were looking at all of the answer choices. Take the time beforehand to do good annotations and the questions will be a lot easier

C) Wrong: Everything in the beginning of this answer choice is right but when it gets to “one possible cause is preferred over the others” it’s wrong. The first sentence of the last paragraph (line 50) says that all 3 changes are created and nowhere else in the passage does it say one is preferred over the other

Why you would accidentally choose this:
• If you didn’t read carefully and just breezed through this. These types of questions, the answer choices are usually extremely similar to one another and so one minor detail is how they differentiate from each other. Read carefully and read the entire answer choice.

D) Wrong: Everything in the beginning of this answer choice is right but when it gets to “3 similar phenomena” it’s wrong. Nowhere in the passage does it present similar phenomena nor does it discuss the similarities between the 2. Because of this the answer choice is wrong.

Why you would accidentally choose this:
• If you didn’t read carefully and just breezed through this. These types of questions, the answer choices are usually extremely similar to one another and so one minor detail is how they differentiate from each other. Read carefully and read the entire answer choice

E) Wrong: Everything in the beginning of this answer choice is right but when it gets to “3 similar phenomena” it’s wrong. Nowhere in the passage does it present similar phenomena nor does it discuss the differences between the 2. This answer choice is saying almost the same thing that “D” is saying but instead of talking about the similarities it says differences. Nowhere in here was there any other comparison so both of these answer choices are wrong.

Question 5:
It can be most reasonably inferred from the passage that

MBT Question: Inferred

What we’re looking for: Something that can be proven from the passage, there are many different directions this could go so it’s difficult to pre-phrase but the passage MUST be able to prove out the answer choice we choose

Answer Choices:
A) Wrong: We don’t know this the only thing the author claims is that authoritarian rulers are pressured to institute democratic reforms. This is outside the scope and therefore we can’t make any statements about it.

Why you might accidentally choose this:
• If you didn’t understand that the answer choice is referring to a specific situation that the passage doesn’t include. If you read it as “many authoritarian rulers will eventually institute democratic reforms” then yes that would be right, but with the qualifier “even if not pressured to do so” that renders this outside of the scope.

B) Wrong: We don’t know when citizen dissatisfaction is highest, so we can’t conclude anything about this either.

Why you might accidentally choose this:
• If you bring in outside information to answer this question then it could be reasonable to assume that citizen dissatisfaction is highest when it would be first imposed. However, this isn’t our world that we’re talking about, we’re concerned about this fake world so we can’t bring in outside information.

C) Wrong: This is similar to “B” in that we can’t conclude anything about when the support is highest for authoritarian regimes. The only thing we know is that the more success the regime has the more likely the regime is to fail, i.e. popular support is lowest when conditions are high.

Why you might accidentally choose this:
• If you mistake logical opposites with real world opposites. If you try to take the contrapositive of what we know from above you would get “popular support is not lowest when conditions are not high”. Note that this is not saying that popular support is highest when conditions are low. Not low could mean high, medium, average or any other part of the spectrum while not high could mean average, semi-low, or zero it’s just everything that isn’t high. Don’t fall for this TRAP ANSWER CHOICE

D) Correct: This is what we’re looking for because that’s the basis of our author’s argument. The last 2 sentences of the passage hint at this (51-60). It talks about people having more time to reflect on the circumstances in which they live and it causes the regime to end up failing because people realize that they want democratic reforms. So if the society doesn’t want the authoritarian regime then they have the power to change and therefore cause the ruler’s to have to change to maintain their power.

Why you might accidentally NOT chose this:
• This is another answer in which it doesn’t jump out at you because it’s very subtle and if you didn’t read and annotate correctly you wouldn’t have picked up on what it was saying. Read carefully and comprehend not understand what you’re reading and you’ll be fine.

E) Wrong: There is nothing in the passage that talk about human rights abuses being the only objectionable aspect of authoritarian regimes. You can’t add anything to the passage you simply have to use what the passage gave you to push out an answer. This isn’t supported by the passage and therefore it is wrong.

Why you might accidentally choose this answer choice:
• Anytime you get to answer choice E you need to be very careful. Testmakers understand the psychology of test takers and how when they’ve gotten to E they’ve already invested time into the question so don’t think that just because there are some elements of things that you remember that you need to choose that answer choice. Invest your time early on and you won’t waste it later on when you get to answer choice time sinks.

Question 6:
Given the information in the passage, authoritarian rulers who institute democratic reforms decide to do so on the basis of which one of the following principles?

What we’re looking for:
• We want an answer choice that says that rulers are going to do whatever they can to maintain as much power for the longest amount of time.

A) Wrong: The article talks about them wanting to maintain their power and nowhere does it mention that they should make an exception “if the health of the nation requires it”.

Why you might accidentally choose this answer choice:
• This is a “feel good answer”, it’s what we wish the world were like and if you bring that information/mentality to the test you’re going to get burned. Don’t bring in any outside information because you’ll be exploited.

B) Wrong: Nowhere in the passage does it mention rulers really caring about the amount of personal freedom their citizens have. If it’s not in the passage then it’s not a principle that is supported.

Why you might accidentally choose this answer choice:
• This is a “feel good answer”, similar to “A” it’s what we wish the world were like and if you bring that information/mentality to the test you’re going to get burned. Don’t bring in any outside information because you’ll be exploited.

C) Wrong: Nowhere in the passage does it mention the rulers wanting to neither transition to democracy nor does it mention that want to transition quickly or efficiently. They are only doing it to keep as much power for themselves as possible.

Why you might accidentally choose this answer:
• This is a “feel good answer”, similar to “A” and “B” it’s what we wish the world were like and if you bring that information/mentality to the test you’re going to get burned. Don’t bring in any outside information because you’ll be exploited.

D) The ruler’s in this passage aren’t concerned about the long-term health of the nation’s economy only maintaining their power for as long as possible. And even if they were concerned about ensuring the long-term health of the nation’s economy the passage doesn’t leave out the possibility of this regime having economic success. The irony is that in lines (50-60) it elaborates that the more success the authoritarian regime has, the more likely it will fall.

Why you might accidentally choose this answer:
• This is a “feel good answer”, similar to “A”, “B”, and “C”I t’s what we wish the world were like and if you bring that information/mentality to the test you’re going to get burned. Don’t bring in any outside information because you’ll be exploited.

E) Correct: This is exactly what we’re looking for; we wanted an answer choice that allows for the rulers to maintain their power for as long as possible. That is why they’re conceding to undergo democratic reform because at least they maintain as much power as long as possible.

Why you might accidentally NOT choose this answer choice:
• Like we’ve said anytime you get to answer choice “E” you should be fully aware of any traps but that doesn’t mean that it isn’t the right answer. If you fell for one of the “feel good answers above” and didn’t read any other answer choices to try and save time then you would have missed this. Make sure you read all of the answer choices and you’ll be fine.
Sign In or Register to comment.